0

A 30 year old lady presented with pulmonary thromboembolism and is found to have APLA. She was given fondaparinux and warfarin and then maintained on warfarin. The appropriate target INR is:

a. 2.0

b. 2.5

c. 3.0

d. 3.5

Pramiti Kumari Answered question August 25, 2024